Helppppp thxxxxxxxxxx

Helppppp Thxxxxxxxxxx

Answers

Answer 1

Answer:

F. [tex] \frac{3}{2} [/tex]

Step-by-step explanation:

[tex] \frac{a + 2b}{b} = \frac{7}{2} [/tex]

Cross multiply:

7b= 2(a +2b)

Expand:

7b= 2a +4b

Bring all common variables to 1 side:

7b -4b= 2a

3b= 2a

divide by 2 on both sides:

[tex] \frac{3}{2} b = a[/tex]

divide by b on both sides:

[tex] \frac{3}{2} = \frac{a}{b} \\ \frac{a}{b} = \frac{3}{2} [/tex]


Related Questions

Lydia drives from city a to city b to transport goods. her return speed is 3 times her departure speed and she takes 40 minutes less on her return trip. how long did her departure trip take?

Answers

Answer:

1 hour

Step-by-step explanation:

Hello, let's say that her departure trip takes t in minutes, as her return speed is 3 times her departure speed, she took t/3 for the return and we know that this 40 minutes less, so we can write.

t/3=t-40

We can multiply by 3

t = 3t -40*3 = 3t - 120

This is equivalent to

3t -120 = t

We subtract t

2t-120 = 0

2t = 120

We divide by 2

t = 120/2 = 60

So this is 60 minutes = 1 hour.

Thank you.

in need of assistance answers are greatly appreciated thank you for your time and effort

Answers

Answer:

x = (h+g)/-f

Step-by-step explanation:

-fx-g = h

Add g to each side

-fx-g+g = h+g

-fx = h+g

Divide each side by -f

-fx/-f = (h+g)/-f

x = (h+g)/-f

Solve for x: −3x + 3 −1 b. x −3

Answers

Answer:

2/3

Step-by-step explanation:

Your −3x + 3 −1 is not an equation and thus has no solution.

If, on the other hand, you meant

−3x + 3 = 1

then -3x = -2, and  x = 2/3

Which relation is a function?

Answers

The number two is a function

First rule of function: for each element of A there is one and only one element of B

For example, in the first one -5 is "collegated" to -2 and 3. So this isn't a function.

Naturally,  every element of B can have more element of A

Michelle is 7 years older than her sister Joan, and Joan is 3 years younger than their brother Ryan. If the sum of their ages is 64, how old is Joan?

16
22
18
19

Answers

Answer:

(C) 18

Step-by-step explanation:

We can create a systems of equations. Assuming [tex]m[/tex] is Michelle's age, [tex]j[/tex] is Joan's age, and [tex]r[/tex] is Ryan's age, the equations are:

[tex]m = j + 7[/tex]

[tex]j = r-3[/tex]

[tex]m+j+r = 64[/tex]

We can use substitution, since we know the "values" of m and j.

[tex](j+7)+(r-3)+r = 64\\(j+7)+(2r-3)=64\\2r + j + 4 = 64\\2r + j = 60\\\\[/tex]

[tex]r = 21, j = 18[/tex]

So we know that Joan is 18 years old.

Hope this helped!

A special mixed-nut blend at a store cost $1.35 per lb, and in 2010 the blend cost $1.83 per lb. Let y represent the cost of a pound of the mixed-nut blend x years after 2005. Use a linear equation model to estimate the cost of a pound of the mixed-nut blend in 2007.

Answers

Answer:

y = $1.542 per lb

Step-by-step explanation:

given data

mixed-nut blend store cost  2005 = $1.35 per lb

blend cost in 2010 = $1.83 per lb

solution

we consider here y = cost of a pound

and x year = after 2005

we will use here linear equation model

so

[tex]\frac{y - 1.35}{1.83-1.35} = \frac{x-10}{5 - 0}[/tex]    .........................1

solve it we get

5y - 6.75 = .48 x

so

at 2007 year here x wil be 2

so

[tex]y = \frac{0.48 \times 2 + 6.75}{5}[/tex]  

solve it we get

y = $1.542 per lb

Question 18 i will maek the brainliest:)

Answers

Answer:

Median: 14.6, Q1: 6.1, Q3: 27.1, IR: 21, outliers:  none

Step-by-step explanation:

Step 1: order the data from the least to the largest.

2.8, 3.9, 5.3, 6.1, 6.5, 7.1, 12.5, 14.6, 16.4, 16.4, 20.8, 27.1, 28.1, 30.9, 53.5

Step 2: find the median.

The median is the middle value, which is the 8th value in the data set.

2.8, 3.9, 5.3, 6.1, 6.5, 7.1, 12.5, [14.6,] 16.4, 16.4, 20.8, 27.1, 28.1, 30.9, 53.5

Median = 14.6

Step 2: Find Q1,

Q1 is the middle value of the lower part of the data set that is divided by the median to your left.

2.8, 3.9, 5.3, (6.1), 6.5, 7.1, 12.5, [14.6], 16.4, 16.4, 20.8, 27.1, 28.1, 30.9, 53.5

Q1 = 6.1

Step 3: find Q3.

Q3 is the middle value of the upper part of the given data set.

2.8, 3.9, 5.3, 6.1, 6.5, 7.1, 12.5, [14.6], 16.4, 16.4, 20.8, (27.1), 28.1, 30.9, 53.5

Q3 = 27.1

Step 4: find interquartile range (IR)

IR = Q3 - Q1 = [tex] 27.1 - 6.1 = 21 [/tex]

Step 5: check if there is any outlier.

Formula for checking for outlier = [tex] Q1 - 1.5*IR [/tex]

Then compare the result you get with the given values in the data set. Any value in the data set that is less than the result we get is considered an outlier.

Thus,

[tex] Q1 - 1.5*IR [/tex]

[tex]6.1 - 1.5*21 = -25.4[/tex]

There are no value in the given data set that is less than -25.4. Therefore, there is no outlier.

Julissa gave out an equal number of oranges to each of the 6 apartments on her floor. if she gave each apartment 5 oranges, how many oranges did Julissa give out in all?

Answers

julissa gave equal oranges in 6 apartments

she gave each apartment 5 oranges

so total no. of oranges are = 6×5 = 30

Answer:

D. 30

Step-by-step explanation:

What is the simplified form of x minus 5 over x squared minus 3x minus 10⋅ x plus 2 over x squared plus x minus 12 ? (6 points) Select one: a. 1 over the quantity x minus 3 times the quantity x plus 4 b. 1 over the quantity x minus 3 times the quantity x plus 2 c. 1 over the quantity x plus 4 times the quantity x minus 5 d. 1 over the quantity x plus 2 times the quantity x minus 5

Answers

Answer:

  [tex]\ \text{a. }\quad\dfrac{1}{(x-3)(x+4)}[/tex]

Step-by-step explanation:

Maybe you want the product ...

  [tex]\dfrac{x-5}{x^2-3x-10}\cdot\dfrac{x+2}{x^2+x-12}=\dfrac{x-5}{(x-5)(x+2)}\cdot\dfrac{x+2}{(x-3)(x+4)}\\\\=\boxed{\dfrac{1}{(x-3)(x+4)}}[/tex]

__

Numerator factors of (x-5) and (x+2) cancel those in the denominator.

Nicole ordered a volleyball for $9.75

Answers

That’s Nice!!

I don’t think you copied the WHOLE ENTIRE question. I would suggest resubmitting the question so we can help you with it!!

Have a great day and stay safe and positive!!

Answer:

the other person is right

you should try putting the WHOLE question

Step-by-step explanation:

A slope triangle for line l is shown on the graph below. If the
slope of the line is 4/3 what is the value of w?

Answers

Answer:

9

Step-by-step explanation:

What we have to note is that the slope of a line is rise/run. This means that the amount of y change in that line is 4, and the amount of x change is 3.

We can now use a proportion to find the value of w.

[tex]\frac{4}{3} = \frac{12}{x}[/tex]

Cross multiply:

[tex]12\cdot36 = 36\\\\36\div4=9[/tex]

Hope this helped!

Answer: 9

Step-by-step explanation:

Suppose that 200 students are randomly selected from a local college campus to investigate the use of cell phones in classrooms. When asked if they are allowed to use cell phones in at least one of their classes, 40% of students responded yes. Using these results, with 95% confidence, the margin of error is 0.068. How would the margin of error change if the sample size increased from 200 to 400 students?

Answers

Answer:

It would change to 0.04802

Step-by-step explanation:

from this question we have that n became 400

40% of 400

= 160

p* = 160/400

= 0.4

1 - p* =

= 1 - 0.4

= 0.6

at confidence level,

1 - 0.95

= 0.05

alpha/2 = 0.025

z= 1.96

margin of error. E

= 1.96 x √[(0.4 x 0.6)/400]

= 1.96 x 0.0245

= 0.04802

M.E = 0.04802

Solve systems of equations 15 points NOT CLICKBAIT!!! -6y+11y= -36 -4y+7x= -24

Answers

Answer:

x = -264/35

y = -36/5

Step-by-step explanation:

-6y + 11y = -36

-4y + 7x = -24

Solve for y in the first equation.

-6y + 11y = -36

Combine like terms.

5y = -36

Divide both sides by 5.

y = -36/5

Plug y as -36/5 in the second equation and solve for x.

-4(-36/5) + 7x = -24

Expand brackets.

144/5 + 7x = -24

Subtract 144/5 from both sides.

7x = -264/5

Divide both sides by 7.

x = -264/35

Answer: -264/35

Step-by-step explanation:

i did my work on a calculator

if 2x-7 is 5 more than x+4, what is the value of 3x+5

Answers

Answer:

  53

Step-by-step explanation:

Let's start with the given relation:

  2x -7 = (x+4) +5

  x = 16 . . . . . . . . . add 7-x

  3x +5 = 3(16) +5 = 53 . . . . . multiply by 3 and add 5

The value of 3x+5 is 53.

Classify the expression: 5x + 3x^2 − 7x^3 + 2
A. Linear Expression
B. Quadratic Expression C. Cubic Expression
D. Quartic Expression

Answers

Answer:

C. Cubic expression.

Step-by-step explanation:

The highest exponent is 3 ( in the term 7x^3) so it is cubic.

Answer:

C. Cubic Expression.

Step-by-step explanation:

5x + 3x^2 - 7x^3 + 2

= 3x^2 - 7x^3 + 5x + 2

= -7x^3 + 3x^2 + 5x + 2

The highest value of exponent in the equation is 3.

For a linear expression, the highest exponent is 1.

For a quadratic expression, the highest exponent is 2.

For a cubic expression, the highest exponent is 3.

For a quartic expression, the highest exponent is 4.

So, this is C. Cubic Expression.

Hope this helps!

Find the intervals on which f is increasing and the intervals on which it is decreasing. f(x)=-2cos^(2)x

Answers

Answer:

Increasing

0°≤x≤180°

Decreasing

180°≤x≤360°

Need Help
Please Show Work​

Answers

Answer:

1= 65 degrees

2=115 degrees

3=115 degrees

Step-by-step explanation: supplementary angles where 115 + x = 180 so go backwards by 180 - 115=65 to find corresponding angles. Angle 3 is also corresponding with the given angle of 115. Angle 2 is opposite the 115 so they have to be equal

The monthly cost (in dollars) of water use is a linear function of the amount of water used (in hundreds of cubic feet, HCF). The cost for using 15 HCF of water is 32.84, and the cost for using 43 HCF is 79.04. What is the cost for using 36 HCF of water?

Answers

Answer:

67.49

Step-by-step explanation:

Let the number of HCF be x.

Let the cost be y.

You are given 2 points of a line: (15, 32.84) and (43, 79.04).

Now we find the equation of the line that passes through those points.

y - y1 = m(x - x1)

y - 32.84 = [(79.04 - 32.84)/(43 - 15)](x - 15)

y - 32.84 = (46.2/28)(x - 15)

y - 32.84 = 1.65(x - 15)

y = 1.65x - 24.75 + 32.84

y = 1.65x + 8.09

Now we let x = 36 and solve for y.

y = 1.65(36) + 8.09

y = 67.49

Each leg of a 45°-45°-90° triangle measures 12 cm.
What is the length of the hypotenuse?
Z
х
45°
45°
O 6 cm
12 cm
12 cm
O 672 cm
O 12 cm
O 122 cm

Answers

Answer:

The legs are 12 cm each, so the hypotenuse is

√(144+144)=12√2

Step-by-step explanation:

Applying the Pythagorean Theorem, the length of the hypotenuse is:  12√2 cm.

The Pythagorean TheoremWhere, a and b are two legs of a right triangle, and c is the hypotenuse, the Pythagorean Theorem states that, c² = a² + b².

Given the two legs of the right triangle to be 12 cm

Therefore:

c² = 12² + 12².

c² = 288

c = √288

c = 12√2 cm

Therefore, applying the Pythagorean Theorem, the length of the hypotenuse is:  12√2 cm.

Learn more about, the Pythagorean Theorem on:

https://brainly.com/question/654982

Jaclyn is one-fourth of a foot taller than John. John is 31/6 feet tall. How many feet tall is Jaclyn

Answers

Answer:

5 5/12

Step-by-step explanation:

31/6 feet + 1/4 foot

= 31/6 + 1/4

= [(31 * 4) / 6 * 4] + [(1 * 6) / 4 * 6]

=  [ 124/24 ] + [ 6/24 ]

= (124 + 6) / 24

= 130 / 24

= 5 10/24

= 5 5/12

Hope this helps!  Tell me if I'm wrong!

Find the area of the irregularly-shaped hexagon below

Answers

let each box length be 1

for white triangle

area = ½bh

=½(4)(2)

=4

for orange triangle

area=½(2)(3)

=3

for blue marked boxes

each of the box

area=l²

=(1)²

=1

there are 16 boxes

so the total area will be 16

total area of the hexagon = 4+3+16

=23 square units

[tex]A_1=\dfrac{1}{2}(3+5)\cdot 3=12\\A_2=1\cdot5=5\\A_3=\dfrac{1}{2}(5+1)\cdot 2=6[/tex]

So the area of the whole shape is [tex]12+5+6=23[/tex]

Answer two questions about Equations A and B:
A. 2x-1=5x
B. -1=3x

1) How can we get Equation B from Equation A?

Choose 1 answer:

Add/subtract the same quantity to/from both sides

Add/subtract a quantity to/from only one side

Rewrite one side (or both) by
combining like terms

Rewrite one side (or both) using the distributive property

NEXT QUESTION
based on the previous answer, are the equations equivalent? In other words, do they have the same solution?

A. Yes
B. No

Answers

Answer:

B: Add/subtract the same quantity to/from both sides

Next Question: Yes

Step-by-step explanation:

thats what the answer is dunno what else to tell you lol

Algebraic equations are mathematical equations that contain unknown variables.

To get Equation B from Equation A, we add/subtract the same quantity to/from both sides. Option A is the correct option. Equation A is equivalent to Equation B

Question 1: We are given equation A as:

2x - 1 = 5x .............Equation A

To get Equation B from A, we would subtract 2x from both sides of the equation.

2x - 2x - 1 = 5x - 2x

- 1 = 3x This is Equation B

Question 2: Based on the previous answer,

2x - 1 = 5x  is equal to  -1 = 3x.

Hence, both Equation A and Equation B are equivalent expressions.

Therefore,

To get Equation B from Equation A, we add/subtract the same quantity to/from both sides. Option A is the correct option.Equation A is equivalent to Equation B

To learn more, visit the link below:

https://brainly.com/question/22299566

Solve for x -3x-3=-3(x+1)

Answers

Step-by-step explanation:

[tex] - 3x - 3 = - 3(x + 1) \\ - 3x - 3 = - 3x - 3 \\ - 3x + 3x = - 3 + 3 \\ 0 = 0[/tex]

Step 1: Use 3 to open the bracket

Step 2 : Collect like terms and simplify

Answer = 0

tan inverse 1/4 +tan inverse 2/7 = 1/2 cos inverse 3/5​

Answers

Answer:

The equation is always false

Step-by-step explanation:

arctan1/4+arctan2/7=1/2arccos3/5

0.24497866+0.27829965=1/2(0.92729521)

0.52327832                 =0.46364760

not equivalent and will never be.

Please Help
Function 1 is defined by the equation: p=r+7
Function 2 is defined by the table shown in the image below
Which function has a greater slope, function 1 or function 2?

Answers

Answer:

The slope of Function 2 (m=1.1) is greater than the slope of Function 1 (m=1).  

Step-by-step explanation:

First, note that p is essentially the y and that r is the x. Thus, to make this easier to see, convert p to y and r to x. Thus:

[tex]y=x+7[/tex]

From the above equation, we can determine that the slope is 1. Thus, the slope of Function 1 is 1.

To find the slope of the table, simply use the slope formula. Use any two points. I'm going to use the points (0,8) and (10,19). Let (0,8) be x₁ and y₁, and (10,19) be x₂ and y₂. Therefore:

[tex]m=\frac{y_2-y_1}{x_2-x_1}=\frac{19-8}{10-0}=11/10=1.1[/tex]

Thus, the slope of Function 2 is 1.1.

1.1 is greater than 1.

Thus, the slope of Function 2 is greater than the slope of Function 1.

Answer:

Function 2 has the greater slope

Step-by-step explanation:

7. Suppose that y varies inversely with x. Write an equation for the inverse variation,
y = 4 when x = 6
A
у
x =
2
B
х
y =
24
с
24
y =
OD y = 2x

Answers

Answer:

The answer is

[tex]y = \frac{24}{x} [/tex]

Step-by-step explanation:

The statement

y varies inversely with x is written as

[tex]y = \frac{k}{x} [/tex]

where k is the constant of proportionality

To find k substitute the values of x and y into the equation

From the question

y = 4

x = 6

We have

[tex]4 = \frac{k}{6} [/tex]

Cross multiply

k = 4 × 6

k = 24

So the formula for the variation is

[tex]y = \frac{24}{x} [/tex]

Hope this helps you

Answer: 5

Step-by-step explanation:

The price of a technology stock was $ 9.56 yesterday. Today, the price rose to $ 9.69 . Find the percentage increase. Round your answer to the nearest tenth of a percent.

Answers

Answer and Step-by-Step explanation:

% increase = 100 x [(new price) - (original price)] / (original price)] = 100 (9.67 - 9.56) / 9.56

% increase ≅ 1.2% (to the nearest tenth)

h(x) = -x² + 3x + 10

Answers

Answer:

x = 5 or x = -2 or 3 - 2 x (derivative)

Step-by-step explanation:

Solve for x over the real numbers:

-x^2 + 3 x + 10 = 0

Multiply both sides by -1:

x^2 - 3 x - 10 = 0

x = (3 ± sqrt((-3)^2 - 4 (-10)))/2 = (3 ± sqrt(9 + 40))/2 = (3 ± sqrt(49))/2:

x = (3 + sqrt(49))/2 or x = (3 - sqrt(49))/2

sqrt(49) = sqrt(7^2) = 7:

x = (3 + 7)/2 or x = (3 - 7)/2

(3 + 7)/2 = 10/2 = 5:

x = 5 or x = (3 - 7)/2

(3 - 7)/2 = -4/2 = -2:

Answer: x = 5 or x = -2

____________________________________

Find the derivative of the following via implicit differentiation:

d/dx(H(x)) = d/dx(10 + 3 x - x^2)

Using the chain rule, d/dx(H(x)) = ( dH(u))/( du) ( du)/( dx), where u = x and d/( du)(H(u)) = H'(u):

(d/dx(x)) H'(x) = d/dx(10 + 3 x - x^2)

The derivative of x is 1:

1 H'(x) = d/dx(10 + 3 x - x^2)

Differentiate the sum term by term and factor out constants:

H'(x) = d/dx(10) + 3 (d/dx(x)) - d/dx(x^2)

The derivative of 10 is zero:

H'(x) = 3 (d/dx(x)) - d/dx(x^2) + 0

Simplify the expression:

H'(x) = 3 (d/dx(x)) - d/dx(x^2)

The derivative of x is 1:

H'(x) = -(d/dx(x^2)) + 1 3

Use the power rule, d/dx(x^n) = n x^(n - 1), where n = 2.

d/dx(x^2) = 2 x:

H'(x) = 3 - 2 x

Simplify the expression:

Answer:  = 3 - 2 x

Need help please will mark brainliest

Answers

Step-by-step explanation:

Maximum = 62

Median = (34+37+39+32+48+45+53+62+58+61+60+41)/12= 47.5≈48

quartile

In increasing order

32, 34, 37, 39, 41, 45, 48, 53, 58, 60, 61, 62

Upper quartile= (58+60)/2 = 59

Lower quartile= (37+39)/2 = 38

Minimum= 32

Which of the following is NOT a requirement of testing a claim about two population means when 1 and 2 are unknown and not assumed to be​ equal? Choose the correct answer below. A. The two samples are dependent. B. Both samples are simple random samples. C. Either the two sample sizes are large ​(30 and ​30) or both samples come from populations having normal​ distributions, or both of these conditions are satisfied. D. The two samples are independent.

Answers

Answer:

b

Step-by-step explanation:

Other Questions
What is the value of x to the nearest tenth? For a particular reaction at 235.8 C, G=936.92 kJ/mol , and S=513.79 J/(molK) . Calculate G for this reaction at 9.9 C. What groups success among mainstream audiences led to the creation of the white rap group The Beastie Boys? n urn contains 3 red balls, 9 green, 2 yellow, 2 orange, and 4 purple balls. Two balls aredrawn, one at a time with replacement. Find the probability of drawing a green ball and an orangeball. The Acme Company lost 650 hours due to accidents on the job in the first quarter of the year. The average hourly wage of the employees who contributed to the lost hours was $15.00. The company benefits add 25% to the wages. Using the cost-of-lost-hours formula (employee hours lost X average loaded labour rate = cost), calculate the direct cost of the lost work hours. Discuss the approach you would use to estimate the hidden costs that would also be associated with the lost hours.Your answer must be at least 200 words in length. Will use a variety of methods to check your work for authenticity and make sure its your own Summary of Possible Weather and Associated Aviation Impacts for Geographic/Topographic Categories Common in the Western United States. Geographic/Topographic Descriptive Summary of Potential Aviation ImpactsCategory of a Possible Weather That Could Impact Based on Weatherof Airport Location Aviation OperationsAlong the US West coast, with steep mountains to the east(An example of this category is Santa Barbara Airport, located on the Southern California Coast, at an elevation of 10 feet).Within a valley in elevated terrain surrounded by high mountains (An example of this category is Friedman Memorial Airport, located in Central Idaho, at an elevation of 5300 feet).In elevated terrain on the leeside of high mountains (An example of this category is Northern Colorado Regional Airport, located in northern Colorado, at an elevation of 5000 feet, on the leeside of the Rocky mountains). 100 cm^3 of oxygen diffuses through a Porous in 3second how long will it take 150 cm^3 of sulphur (iv) oxide diffuse through the same pot? ( oxygen= 16 sulphur = 32) Tina's age is 4 years less than 3 times her niece's age. If her niece's age is x years, which of the following expressions best shows Tina's age? x 4 4x 3 3x 4 4 3x Lisa rides the train to work and always gets off at Stop A. One Saturday she has to go into town, and she rides the same train she takes to work. She is supposed to get off at Stop F, but she starts talking to her mother on the phone and then gets off at Stop A. What does this tell us about unconscious processing? 1. What does it mean to be an American? show that the point p(-6,2), Q(1,7) and R(6,3) are the vertices of scalene triangle Astrid is in charge of building a new fleet of ships. Each ship requires 404040 tons of wood, and accommodates 300300300 sailors. She receives a delivery of 444 tons of wood each day. The deliveries can continue for 100100100 days at most, afterwards the weather is too bad to allow them. Overall, she wants to build enough ships to accommodate at least 210021002100 sailors. In "The Grapes of Wrath", Mae wants the man to buy a sandwich and not treat the diner as a grocery store. What is the second part of the five-and-five approach called? Henry maneuver Heimlich maneuver abdominal maneuver back maneuver A company makes nylon and canvas backpacks. The profit on a nylon backpack is $3 and the profit on a canvas backpack is $10. How many backpacks must the company sell to make a profit of more than $250? Write a linear inequality that describes the situation. HELP ME I NEED IT [100 POINTS ]Match each Southeast Asian nation to its description. Sri Lanka The Philippines Myanmar ruled by a military government called a junta arrowRight elected Corazon Aquino as its president in 1986 after the dictatorship of Ferdinand Marcos arrowRight experienced a civil war between the Sinhalese and the Tamil after independence famous comparative and superlative form PLEASE ANSWER QUICKLY ASAP COMPLETE QUESTION B how to write this in number form The difference of 9 and the square of a number